$E$ is a nonempty subset of an ordered set. If $alpha$ is a lower bound of $E$ and $beta$ is an upper bound, then $alpha leq beta$.

The name of the pictureThe name of the pictureThe name of the pictureClash Royale CLAN TAG#URR8PPP











up vote
2
down vote

favorite












I think I may be citing the law of transitivity incorrectly in this proof.



Theorem. Let $E$ be a nonempty subset of an ordered set. Suppose $alpha$ is a lower bound of $E$ and $beta$ is an upper bound of $E$. Prove that $alpha leq beta$.



Proof. Since $alpha$ is a lower bound of $E$, and $E$ is nonempty, we have that $forall x in E, x geq alpha$. Similarly, since $beta$ is an upper bound, we have $forall x in E, x leq beta$. Stringing these together yields $forall x in E, alpha leq x leq beta$, which by the transitivity property of ordered sets yields $alpha leq beta$.



Here are my questions on this:



(a) Is it a fair assumption that a subset of an ordered set is also ordered? I wasn't sure whether I ought to 'prove' this first, though it seems almost too trivial to prove.



(b) I use transitivity a bit differently from Rudin's definition, based on past formulations I've seen. He seems to define this as $x < y wedge y < z implies x < z$, i.e., with strict inequalities. Should I break this into cases, wherein I consider all possible cases of strict inequalities as well as equalities between $alpha$, $x$, and $beta$? Or is this a natural extension of this property?



Thanks.







share|cite|improve this question






















  • It seems good. Regarding your second question, it is a natural extension.
    – Dog_69
    Aug 18 at 17:15














up vote
2
down vote

favorite












I think I may be citing the law of transitivity incorrectly in this proof.



Theorem. Let $E$ be a nonempty subset of an ordered set. Suppose $alpha$ is a lower bound of $E$ and $beta$ is an upper bound of $E$. Prove that $alpha leq beta$.



Proof. Since $alpha$ is a lower bound of $E$, and $E$ is nonempty, we have that $forall x in E, x geq alpha$. Similarly, since $beta$ is an upper bound, we have $forall x in E, x leq beta$. Stringing these together yields $forall x in E, alpha leq x leq beta$, which by the transitivity property of ordered sets yields $alpha leq beta$.



Here are my questions on this:



(a) Is it a fair assumption that a subset of an ordered set is also ordered? I wasn't sure whether I ought to 'prove' this first, though it seems almost too trivial to prove.



(b) I use transitivity a bit differently from Rudin's definition, based on past formulations I've seen. He seems to define this as $x < y wedge y < z implies x < z$, i.e., with strict inequalities. Should I break this into cases, wherein I consider all possible cases of strict inequalities as well as equalities between $alpha$, $x$, and $beta$? Or is this a natural extension of this property?



Thanks.







share|cite|improve this question






















  • It seems good. Regarding your second question, it is a natural extension.
    – Dog_69
    Aug 18 at 17:15












up vote
2
down vote

favorite









up vote
2
down vote

favorite











I think I may be citing the law of transitivity incorrectly in this proof.



Theorem. Let $E$ be a nonempty subset of an ordered set. Suppose $alpha$ is a lower bound of $E$ and $beta$ is an upper bound of $E$. Prove that $alpha leq beta$.



Proof. Since $alpha$ is a lower bound of $E$, and $E$ is nonempty, we have that $forall x in E, x geq alpha$. Similarly, since $beta$ is an upper bound, we have $forall x in E, x leq beta$. Stringing these together yields $forall x in E, alpha leq x leq beta$, which by the transitivity property of ordered sets yields $alpha leq beta$.



Here are my questions on this:



(a) Is it a fair assumption that a subset of an ordered set is also ordered? I wasn't sure whether I ought to 'prove' this first, though it seems almost too trivial to prove.



(b) I use transitivity a bit differently from Rudin's definition, based on past formulations I've seen. He seems to define this as $x < y wedge y < z implies x < z$, i.e., with strict inequalities. Should I break this into cases, wherein I consider all possible cases of strict inequalities as well as equalities between $alpha$, $x$, and $beta$? Or is this a natural extension of this property?



Thanks.







share|cite|improve this question














I think I may be citing the law of transitivity incorrectly in this proof.



Theorem. Let $E$ be a nonempty subset of an ordered set. Suppose $alpha$ is a lower bound of $E$ and $beta$ is an upper bound of $E$. Prove that $alpha leq beta$.



Proof. Since $alpha$ is a lower bound of $E$, and $E$ is nonempty, we have that $forall x in E, x geq alpha$. Similarly, since $beta$ is an upper bound, we have $forall x in E, x leq beta$. Stringing these together yields $forall x in E, alpha leq x leq beta$, which by the transitivity property of ordered sets yields $alpha leq beta$.



Here are my questions on this:



(a) Is it a fair assumption that a subset of an ordered set is also ordered? I wasn't sure whether I ought to 'prove' this first, though it seems almost too trivial to prove.



(b) I use transitivity a bit differently from Rudin's definition, based on past formulations I've seen. He seems to define this as $x < y wedge y < z implies x < z$, i.e., with strict inequalities. Should I break this into cases, wherein I consider all possible cases of strict inequalities as well as equalities between $alpha$, $x$, and $beta$? Or is this a natural extension of this property?



Thanks.









share|cite|improve this question













share|cite|improve this question




share|cite|improve this question








edited Aug 18 at 20:16









Asaf Karagila♦

294k31410737




294k31410737










asked Aug 18 at 17:06









Matt.P

930414




930414











  • It seems good. Regarding your second question, it is a natural extension.
    – Dog_69
    Aug 18 at 17:15
















  • It seems good. Regarding your second question, it is a natural extension.
    – Dog_69
    Aug 18 at 17:15















It seems good. Regarding your second question, it is a natural extension.
– Dog_69
Aug 18 at 17:15




It seems good. Regarding your second question, it is a natural extension.
– Dog_69
Aug 18 at 17:15










2 Answers
2






active

oldest

votes

















up vote
2
down vote



accepted










For your first question, your ordering actually relies on the ordering on the larger set, since the upper and lower bounds need not be members of the subset $E$ itself! For instance, if the original set is $mathbbR$ and the subset is $(0,1)$, your lower bound could be $0$. So really you rely on the fact that the ordering is defined on the original set.



For your second question, to be completely rigorous, the transitivity of $leq$ follows from transitivity of $<$ and $=$. Personally, I think it is fine to simply leave it as you have, but perhaps your professor wants you to add in additional detail if this is a first course in proofs or something.






share|cite|improve this answer



























    up vote
    5
    down vote













    You're proof is almost correct.



    • The fact that E is nonempty is useless to state that ∀x∈E, x≥α (it would also be the case if E were empty!). Same for ∀x∈E, x≤β. These two relations are the definition of lower and upper bounds, respectively.


    • However, you need the fact that E is nonempty to prove that α≤β, because ∀x∈E,α≤x≤β would be also true if E were empty, but you couldn't argue that α≤β (because in this case α≤x≤β would never happen). A correct statement would be "E is nonempty so ∃x∈E. So α≤x and x≤β so α≤β."


    (a) Yes, you can prove it easily.



    (b) "<" and ">" are not orders because they do not check the reflexivity property of an order






    share|cite|improve this answer


















    • 1




      They can be considered as strict orders. In fact, some books define orders as $<$ instead of $leq$.
      – Dog_69
      Aug 18 at 17:28






    • 1




      Would you mind explaining why they don't satisfy antisymmetry? It seems that $a < b$, for example, cannot coexist with $b < a$. Surely $<$ wouldn't be reflexive, though, whereas $leq$ would be.
      – Matt.P
      Aug 18 at 17:33










    • @Matt.P Oh, very good point. I think he/s wanted to say reflexivity.
      – Dog_69
      Aug 18 at 17:47










    • Yes, my mistake I wanted to say reflexivity. I fixed it, thx
      – Edouardb
      Aug 20 at 5:48










    Your Answer




    StackExchange.ifUsing("editor", function ()
    return StackExchange.using("mathjaxEditing", function ()
    StackExchange.MarkdownEditor.creationCallbacks.add(function (editor, postfix)
    StackExchange.mathjaxEditing.prepareWmdForMathJax(editor, postfix, [["$", "$"], ["\\(","\\)"]]);
    );
    );
    , "mathjax-editing");

    StackExchange.ready(function()
    var channelOptions =
    tags: "".split(" "),
    id: "69"
    ;
    initTagRenderer("".split(" "), "".split(" "), channelOptions);

    StackExchange.using("externalEditor", function()
    // Have to fire editor after snippets, if snippets enabled
    if (StackExchange.settings.snippets.snippetsEnabled)
    StackExchange.using("snippets", function()
    createEditor();
    );

    else
    createEditor();

    );

    function createEditor()
    StackExchange.prepareEditor(
    heartbeatType: 'answer',
    convertImagesToLinks: true,
    noModals: false,
    showLowRepImageUploadWarning: true,
    reputationToPostImages: 10,
    bindNavPrevention: true,
    postfix: "",
    noCode: true, onDemand: true,
    discardSelector: ".discard-answer"
    ,immediatelyShowMarkdownHelp:true
    );



    );













     

    draft saved


    draft discarded


















    StackExchange.ready(
    function ()
    StackExchange.openid.initPostLogin('.new-post-login', 'https%3a%2f%2fmath.stackexchange.com%2fquestions%2f2886916%2fe-is-a-nonempty-subset-of-an-ordered-set-if-alpha-is-a-lower-bound-of-e%23new-answer', 'question_page');

    );

    Post as a guest






























    2 Answers
    2






    active

    oldest

    votes








    2 Answers
    2






    active

    oldest

    votes









    active

    oldest

    votes






    active

    oldest

    votes








    up vote
    2
    down vote



    accepted










    For your first question, your ordering actually relies on the ordering on the larger set, since the upper and lower bounds need not be members of the subset $E$ itself! For instance, if the original set is $mathbbR$ and the subset is $(0,1)$, your lower bound could be $0$. So really you rely on the fact that the ordering is defined on the original set.



    For your second question, to be completely rigorous, the transitivity of $leq$ follows from transitivity of $<$ and $=$. Personally, I think it is fine to simply leave it as you have, but perhaps your professor wants you to add in additional detail if this is a first course in proofs or something.






    share|cite|improve this answer
























      up vote
      2
      down vote



      accepted










      For your first question, your ordering actually relies on the ordering on the larger set, since the upper and lower bounds need not be members of the subset $E$ itself! For instance, if the original set is $mathbbR$ and the subset is $(0,1)$, your lower bound could be $0$. So really you rely on the fact that the ordering is defined on the original set.



      For your second question, to be completely rigorous, the transitivity of $leq$ follows from transitivity of $<$ and $=$. Personally, I think it is fine to simply leave it as you have, but perhaps your professor wants you to add in additional detail if this is a first course in proofs or something.






      share|cite|improve this answer






















        up vote
        2
        down vote



        accepted







        up vote
        2
        down vote



        accepted






        For your first question, your ordering actually relies on the ordering on the larger set, since the upper and lower bounds need not be members of the subset $E$ itself! For instance, if the original set is $mathbbR$ and the subset is $(0,1)$, your lower bound could be $0$. So really you rely on the fact that the ordering is defined on the original set.



        For your second question, to be completely rigorous, the transitivity of $leq$ follows from transitivity of $<$ and $=$. Personally, I think it is fine to simply leave it as you have, but perhaps your professor wants you to add in additional detail if this is a first course in proofs or something.






        share|cite|improve this answer












        For your first question, your ordering actually relies on the ordering on the larger set, since the upper and lower bounds need not be members of the subset $E$ itself! For instance, if the original set is $mathbbR$ and the subset is $(0,1)$, your lower bound could be $0$. So really you rely on the fact that the ordering is defined on the original set.



        For your second question, to be completely rigorous, the transitivity of $leq$ follows from transitivity of $<$ and $=$. Personally, I think it is fine to simply leave it as you have, but perhaps your professor wants you to add in additional detail if this is a first course in proofs or something.







        share|cite|improve this answer












        share|cite|improve this answer



        share|cite|improve this answer










        answered Aug 18 at 17:15









        ThomasGrubb

        9,78011335




        9,78011335




















            up vote
            5
            down vote













            You're proof is almost correct.



            • The fact that E is nonempty is useless to state that ∀x∈E, x≥α (it would also be the case if E were empty!). Same for ∀x∈E, x≤β. These two relations are the definition of lower and upper bounds, respectively.


            • However, you need the fact that E is nonempty to prove that α≤β, because ∀x∈E,α≤x≤β would be also true if E were empty, but you couldn't argue that α≤β (because in this case α≤x≤β would never happen). A correct statement would be "E is nonempty so ∃x∈E. So α≤x and x≤β so α≤β."


            (a) Yes, you can prove it easily.



            (b) "<" and ">" are not orders because they do not check the reflexivity property of an order






            share|cite|improve this answer


















            • 1




              They can be considered as strict orders. In fact, some books define orders as $<$ instead of $leq$.
              – Dog_69
              Aug 18 at 17:28






            • 1




              Would you mind explaining why they don't satisfy antisymmetry? It seems that $a < b$, for example, cannot coexist with $b < a$. Surely $<$ wouldn't be reflexive, though, whereas $leq$ would be.
              – Matt.P
              Aug 18 at 17:33










            • @Matt.P Oh, very good point. I think he/s wanted to say reflexivity.
              – Dog_69
              Aug 18 at 17:47










            • Yes, my mistake I wanted to say reflexivity. I fixed it, thx
              – Edouardb
              Aug 20 at 5:48














            up vote
            5
            down vote













            You're proof is almost correct.



            • The fact that E is nonempty is useless to state that ∀x∈E, x≥α (it would also be the case if E were empty!). Same for ∀x∈E, x≤β. These two relations are the definition of lower and upper bounds, respectively.


            • However, you need the fact that E is nonempty to prove that α≤β, because ∀x∈E,α≤x≤β would be also true if E were empty, but you couldn't argue that α≤β (because in this case α≤x≤β would never happen). A correct statement would be "E is nonempty so ∃x∈E. So α≤x and x≤β so α≤β."


            (a) Yes, you can prove it easily.



            (b) "<" and ">" are not orders because they do not check the reflexivity property of an order






            share|cite|improve this answer


















            • 1




              They can be considered as strict orders. In fact, some books define orders as $<$ instead of $leq$.
              – Dog_69
              Aug 18 at 17:28






            • 1




              Would you mind explaining why they don't satisfy antisymmetry? It seems that $a < b$, for example, cannot coexist with $b < a$. Surely $<$ wouldn't be reflexive, though, whereas $leq$ would be.
              – Matt.P
              Aug 18 at 17:33










            • @Matt.P Oh, very good point. I think he/s wanted to say reflexivity.
              – Dog_69
              Aug 18 at 17:47










            • Yes, my mistake I wanted to say reflexivity. I fixed it, thx
              – Edouardb
              Aug 20 at 5:48












            up vote
            5
            down vote










            up vote
            5
            down vote









            You're proof is almost correct.



            • The fact that E is nonempty is useless to state that ∀x∈E, x≥α (it would also be the case if E were empty!). Same for ∀x∈E, x≤β. These two relations are the definition of lower and upper bounds, respectively.


            • However, you need the fact that E is nonempty to prove that α≤β, because ∀x∈E,α≤x≤β would be also true if E were empty, but you couldn't argue that α≤β (because in this case α≤x≤β would never happen). A correct statement would be "E is nonempty so ∃x∈E. So α≤x and x≤β so α≤β."


            (a) Yes, you can prove it easily.



            (b) "<" and ">" are not orders because they do not check the reflexivity property of an order






            share|cite|improve this answer














            You're proof is almost correct.



            • The fact that E is nonempty is useless to state that ∀x∈E, x≥α (it would also be the case if E were empty!). Same for ∀x∈E, x≤β. These two relations are the definition of lower and upper bounds, respectively.


            • However, you need the fact that E is nonempty to prove that α≤β, because ∀x∈E,α≤x≤β would be also true if E were empty, but you couldn't argue that α≤β (because in this case α≤x≤β would never happen). A correct statement would be "E is nonempty so ∃x∈E. So α≤x and x≤β so α≤β."


            (a) Yes, you can prove it easily.



            (b) "<" and ">" are not orders because they do not check the reflexivity property of an order







            share|cite|improve this answer














            share|cite|improve this answer



            share|cite|improve this answer








            edited Aug 20 at 5:47

























            answered Aug 18 at 17:15









            Edouardb

            480314




            480314







            • 1




              They can be considered as strict orders. In fact, some books define orders as $<$ instead of $leq$.
              – Dog_69
              Aug 18 at 17:28






            • 1




              Would you mind explaining why they don't satisfy antisymmetry? It seems that $a < b$, for example, cannot coexist with $b < a$. Surely $<$ wouldn't be reflexive, though, whereas $leq$ would be.
              – Matt.P
              Aug 18 at 17:33










            • @Matt.P Oh, very good point. I think he/s wanted to say reflexivity.
              – Dog_69
              Aug 18 at 17:47










            • Yes, my mistake I wanted to say reflexivity. I fixed it, thx
              – Edouardb
              Aug 20 at 5:48












            • 1




              They can be considered as strict orders. In fact, some books define orders as $<$ instead of $leq$.
              – Dog_69
              Aug 18 at 17:28






            • 1




              Would you mind explaining why they don't satisfy antisymmetry? It seems that $a < b$, for example, cannot coexist with $b < a$. Surely $<$ wouldn't be reflexive, though, whereas $leq$ would be.
              – Matt.P
              Aug 18 at 17:33










            • @Matt.P Oh, very good point. I think he/s wanted to say reflexivity.
              – Dog_69
              Aug 18 at 17:47










            • Yes, my mistake I wanted to say reflexivity. I fixed it, thx
              – Edouardb
              Aug 20 at 5:48







            1




            1




            They can be considered as strict orders. In fact, some books define orders as $<$ instead of $leq$.
            – Dog_69
            Aug 18 at 17:28




            They can be considered as strict orders. In fact, some books define orders as $<$ instead of $leq$.
            – Dog_69
            Aug 18 at 17:28




            1




            1




            Would you mind explaining why they don't satisfy antisymmetry? It seems that $a < b$, for example, cannot coexist with $b < a$. Surely $<$ wouldn't be reflexive, though, whereas $leq$ would be.
            – Matt.P
            Aug 18 at 17:33




            Would you mind explaining why they don't satisfy antisymmetry? It seems that $a < b$, for example, cannot coexist with $b < a$. Surely $<$ wouldn't be reflexive, though, whereas $leq$ would be.
            – Matt.P
            Aug 18 at 17:33












            @Matt.P Oh, very good point. I think he/s wanted to say reflexivity.
            – Dog_69
            Aug 18 at 17:47




            @Matt.P Oh, very good point. I think he/s wanted to say reflexivity.
            – Dog_69
            Aug 18 at 17:47












            Yes, my mistake I wanted to say reflexivity. I fixed it, thx
            – Edouardb
            Aug 20 at 5:48




            Yes, my mistake I wanted to say reflexivity. I fixed it, thx
            – Edouardb
            Aug 20 at 5:48

















             

            draft saved


            draft discarded















































             


            draft saved


            draft discarded














            StackExchange.ready(
            function ()
            StackExchange.openid.initPostLogin('.new-post-login', 'https%3a%2f%2fmath.stackexchange.com%2fquestions%2f2886916%2fe-is-a-nonempty-subset-of-an-ordered-set-if-alpha-is-a-lower-bound-of-e%23new-answer', 'question_page');

            );

            Post as a guest













































































            Comments

            Popular posts from this blog

            Long meetings (6-7 hours a day): Being “babysat” by supervisor

            Is the Concept of Multiple Fantasy Races Scientifically Flawed? [closed]

            Confectionery